cesenatico 1999

Numeri interi, razionali, divisibilità, equazioni diofantee, ...
Rispondi
Avatar utente
ummagumma
Messaggi: 94
Iscritto il: 22 lug 2007, 11:14

cesenatico 1999

Messaggio da ummagumma »

cesenatico 1999, sesto quesito (senza latex :oops: )
3^k - 1= x^3
avendo appena letto il t. di Fermat credo di fare un pasticcio..cmq:

x^3 + 1 = 0 (mod 3)

x^3 = 2 (mod3)

di qui posso concludere ke l'UNICO x è 2?
se fosse così allora il punto 1 sarebbe dimostrato senza le considerazioni della sol proposta...

inoltre: dimostrare che 3^k - 1= x^n
per lo stesso motivo..posso dire x^n=2^n

2^n = 3^k - 1

posto k pari..k=2h allora:

3^h - 1 = 2^m e 3^h + 1 = 2^q con m+q=n

Sommando: 3^h = 2^(m-1) (1 + 2^(q-m))
l'unica soluzione si ha per m=1 quindi h=1, da cui k=2 e n=3 come trovato in precedenza.

posto k dispari:
2^n = 4 (3^(k - 1) + 3^(k - 2) +....+ 1)

nella parentesi vi sono k-1 potenze di 3 + 1;
essendo k dispari, la somma di un numero pari (k-1) di potenze di 3 è un numero pari ke sommato ad un numero 1 dà ad un numero dispari.

Dunque essendo la quantità in parentesi un numero dispari, non vi sono soluzioni.

Ho finito il delirio, contate ke sono in terza e ho appena iniziato con le olimat
Avatar utente
jordan
Messaggi: 3988
Iscritto il: 02 feb 2007, 21:19
Località: Pescara
Contatta:

Messaggio da jordan »

con le congruenze non trovi mai un'unica soluzione, anzi di solito serve a dimostrare che non ne esiste nessuna..
Avatar utente
julio14
Messaggi: 1208
Iscritto il: 11 dic 2006, 18:52
Località: Berlino

Messaggio da julio14 »

ummagumma ha scritto:di qui posso concludere ke l'UNICO x è 2?
come già diceva jordan, con le congruenze non trovi un unica soluzione: quello che hai dimostrato tu è che se ci sono delle x che soddisfano l'equazione, quelle x elevate al cubo sono congruenti a 2 (mod3) (e quindi anche $ x \cong 2 (mod 3) $
CoNVeRGe.
Messaggi: 98
Iscritto il: 22 ott 2008, 18:51

Messaggio da CoNVeRGe. »

potreste risolvermi questo problema senza l'utilizzo di troppa teoria? (anche con le congruenze va bene, meglio ancora se in diversi modi) :)

lo riformulo in latex:

(a) Determinare tutte le coppie $ \displaystyle (x,k) $ di interi positivi che soddisfano l'equazione

$ \displaystyle 3^k - 1 = x^3 $

(b) Dimostrare che se n è un intero maggiore di 1 e diverso da 3 non esistono coppie $ \displaystyle (x,k) $ di interi positivi che soddisfano l'equazione
$ \displaystyle 3^k - 1 = x^n $
Ultima modifica di CoNVeRGe. il 13 mar 2009, 20:15, modificato 1 volta in totale.
Avatar utente
SkZ
Messaggi: 3333
Iscritto il: 03 ago 2006, 21:02
Località: Concepcion, Chile
Contatta:

Messaggio da SkZ »

buttiamo qualcosa nei ritagli
$ ~3^k=x^3-1=(x-1)(x^2+x+1) $
quindi $ ~x=3^a+1 $ con $ ~0< a<k $
Ultima modifica di SkZ il 13 mar 2009, 20:20, modificato 1 volta in totale.
impara il [tex]~\LaTeX[/tex] e mettilo da par[tex]\TeX~[/tex]

Software is like sex: it's better when it's free (Linus T.)
membro: Club Nostalgici
Non essere egoista, dona anche tu! http://fpv.hacknight.org/a8.php
CoNVeRGe.
Messaggi: 98
Iscritto il: 22 ott 2008, 18:51

Messaggio da CoNVeRGe. »

hai confuso il testo, non è $ \displaystyle 3^k + 1 = x^3 $ ma $ \displaystyle 3^k - 1 = x^3 $
Avatar utente
SkZ
Messaggi: 3333
Iscritto il: 03 ago 2006, 21:02
Località: Concepcion, Chile
Contatta:

Messaggio da SkZ »

buttiamo qualcosa nei ritagli2 (la vendetta :twisted: )
$ ~3^k=x^3+1=(x+1)(x^2-x+1) $
quindi $ ~x=3^a-1 $ con $ ~0< a<k $
impara il [tex]~\LaTeX[/tex] e mettilo da par[tex]\TeX~[/tex]

Software is like sex: it's better when it's free (Linus T.)
membro: Club Nostalgici
Non essere egoista, dona anche tu! http://fpv.hacknight.org/a8.php
pak-man
Messaggi: 313
Iscritto il: 07 giu 2008, 18:19

Messaggio da pak-man »

vediamo se riesco a non scrivere cavolate nonostante l'ora...
$ 3^k-1=x^3 $
$ 3^k=x^3+1=(x+1)(x^2-x+1) $
$ \left\{\begin{array}{llll}x+1=3^a\\x^2-x+1=3^b\\a+b=k\\k>a,b>0\end{array}\right. $
$ \left\{\begin{array}{llll}x=3^a-1\\(3^a-1)^2-(3^a-1)+1=3^b\\a+b=k\\k>a,b>0\end{array}\right. $
Consideriamo la seconda equazione:
$ 3^{2a}-2\cdot3^a+1-3^a+1+1=3^b $
$ 3^{2a}-3^{a+1}+3=3^b $
Per la condizione $ a,b>0 $ possiamo dividere per 3:
$ 3^{2a-1}-3^a+1=3^{b-1} $
Questa equazione è falsa modulo 3 a meno che almeno uno degli esponenti sia zero, e l'unico caso possibile è $ b=1 $. Allora
$ 3^{2a-1}=3^a $
$ 2a-1=a $
$ a=1 $
Quindi l'unica soluzione è $ (x,k)=(2,2) $
Avatar utente
Cassa
Messaggi: 236
Iscritto il: 28 mar 2006, 21:48
Località: Genova

Messaggio da Cassa »

Farò una domanda scema e materialistica...
Ipotesi: In gara faccio questo problema usando la congettura di Catalan dimostrata da Mihailescu (?) e sbroglio il problema in una riga.
E' valido? Cioè, in soldoni, prendo 7?
Tibor Gallai
Messaggi: 1776
Iscritto il: 17 nov 2007, 19:12

Messaggio da Tibor Gallai »

Cassa ha scritto:Ipotesi: In gara faccio questo problema usando la congettura di Catalan dimostrata da Mihailescu (?) e sbroglio il problema in una riga.
E' valido? Cioè, in soldoni, prendo 7?
No, perché l'ipotesi che assumi implica che tu l'abbia fatto nel 1999, anno in cui la congettura di Catalan non era ancora stata dimostrata. Avresti in sostanza barato ed avresti preso 0 punti.
Il motivo è duplice: lo stesso problema non viene riproposto più volte nelle gare singole (quindi un problema del 1999 non è stato e non sarà proposto in altri anni), ed inoltre dal 2002 non ha più senso proporre un problema simile, appunto perché esiste la soluzione che invoca il teorema di Mihailescu.
fph
Site Admin
Messaggi: 3958
Iscritto il: 01 gen 1970, 01:00
Località: in giro
Contatta:

Messaggio da fph »

Cassa ha scritto:Farò una domanda scema e materialistica...
Ipotesi: In gara faccio questo problema usando la congettura di Catalan dimostrata da Mihailescu (?) e sbroglio il problema in una riga.
E' valido? Cioè, in soldoni, prendo 7?
Di solito si cerca di evitare il problema alla radice evitando il più possibile di dare problemi che abbiano questo tipo di... problemi.
E comunque se ti dimentichi di verificare il caso k=1 io sarei più propenso a darti 1.
--federico
[tex]\frac1{\sqrt2}\bigl(\left|\text{loves me}\right\rangle+\left|\text{loves me not}\right\rangle\bigr)[/tex]
Avatar utente
Cassa
Messaggi: 236
Iscritto il: 28 mar 2006, 21:48
Località: Genova

Messaggio da Cassa »

Come immaginavo, grazie :D
Avatar utente
Reginald
Messaggi: 137
Iscritto il: 24 gen 2009, 15:52
Località: Trento

Messaggio da Reginald »

Oh, che bello il mio primo problema risolto (in 2 ore e mezza, ero ancora più scarso di adesso) di teoria dei numeri.
Rispondo a CoNVeRGe e posto due soluzioni.
Risolvo il punto b
$ 3^x-1=n^k $ con k diverso da 3.
Dimostro prima di tutto che k è dispari.
$ n^k\equiv -1\pmod 3 $ per ipotesi. A questo punto n puo essere congrua a 1 o a 2 modulo 3. Se n è congrua a 1, ma k è pari, $ n^2\equiv 1\pmod 3 $ e lo stesso se è congrua a 2 modulo 3 (e k pari). Allora k deve essere dispari. (Anche perchè -1 non è residuo quadratico modulo 3)
Allora posso scrivere questa cosa:
$ 3^x=(n+1)(n^{k-1}-n^{k-2}+n^{k-3}-......-n+1) $.
Allora
$ \begin {cases} 3^i=n+1\\ 3^y=n^{k-1}-....+1\\ \end {cases} $.
Dalla prima ottengo
$ n=3^i-1 $ e sostituendo n nella seconda ottengo$ 3^y=(3^i+1)^{k-1}-(3^i+1)^{k-2}+....-3^i+1+1 $
A questo punto svolgendo le potenze ottengo una cosa del genere:$ 3J+k=3^y $, che implica che k sia multiplo di 3. Dal momento che le uniche soluzioni con k multiplo di 3 implicano k=3, che non è legale per ipotesi, non ci sono soluzioni.

Volendo, dopo aver dimostrato che k è dispari, si poteva procedere così:
$ 3^x=(n+1)(\sum_{i=0}^{k-1}{n^i(-1)^{k-1-i}})\implies n\equiv -1\pmod {3^t} $ allora $ \sum_{i=0}^{k-1}{n^i(-1)^{k-1-i}}\equiv \sum_{i=0}^{k-1}{n^i(n)^{k-1-i}}\equiv 0\pmod {3^t} $,svolgendo i calcoli $ \sum_{i=0}^{k-1}{n^{k-1}}=k*n^{k-1}\equiv 0\pmod {3^t}\implies 3|k $. E quindi l'unica soluzione è k=3 per lo stesso motivo di sopra.
Spero di non aver fatto errori..
Rispondi